LSAT and Law School Admissions Forum

Get expert LSAT preparation and law school admissions advice from PowerScore Test Preparation.

 Anfernee9320
  • Posts: 8
  • Joined: Jan 15, 2020
|
#88577
I am having trouble understanding why the rule is diagrammed as R>NR when the setup does not reflect the order of the variables. In the set up the 6th variable is Rock Classic and the 5th and 7th variables are New Composition.
User avatar
 Dave Killoran
PowerScore Staff
  • PowerScore Staff
  • Posts: 5852
  • Joined: Mar 25, 2011
|
#88578
Hi Anfernee,

Thanks for the message! Two things here:

First, I believe you have a rather old book, since we stopped using the ">" notation in diagramming over 5 years ago. We did this because LSAC started using game setups that thwarted that method; the new method is more flexible and allows for handling more complex games easily. There is also more material that has been added over the years (current version is 746 pages in length). I'd consider getting a current version if possible.

Second, each game has its own specific area in the LG area of this forum, which for this game is at: viewforum.php?f=186. For your question, I specifically discuss the rule you ask about in the setup discussion here: viewtopic.php?f=186&t=11905 (look for it a few posts down). If that doesn't cover it, please let us know.

Thanks!
 Anfernee9320
  • Posts: 8
  • Joined: Jan 15, 2020
|
#88583
Hi Dave,

My apologies I used ">" because I was on my phone earlier and I was having trouble including the arrows. The book I have is the 2020-2021 edition. I will check out the links you provided and let you know if I have any additional questions. Thank you!
User avatar
 Dave Killoran
PowerScore Staff
  • PowerScore Staff
  • Posts: 5852
  • Joined: Mar 25, 2011
|
#88608
Ahh, excellent! I always get worried when I see the > since it usually means the book is missing a lot of info I added. Glad that's not the case here :)
 Anfernee9320
  • Posts: 8
  • Joined: Jan 15, 2020
|
#88627
Hi again,

I looked over the links you provided and I only have one question
  • Does the indicator "Each" make the rule conditional? So every time I see "Each" I should anticipate a conditional statement, specifically a sufficient condition?
Thank you in advance for your response.
User avatar
 Dave Killoran
PowerScore Staff
  • PowerScore Staff
  • Posts: 5852
  • Joined: Mar 25, 2011
|
#88631
Typically yes, because "each" is specifying that when you see a certain occurrence, then there is a characteristic that accompanies all of those occurrence, as in, "Each person in this room has a yellow shirt (person in room :arrow: yellow shirt).

Thanks!
User avatar
 Mswise34
  • Posts: 1
  • Joined: Feb 13, 2022
|
#93736
Linear game #4 : February 1993 (p. 266 on my iPad) sets a condition “ if she operates on Monday, she does not operate on Tuesday”. Question 11. asks “If Doctor Yamata operates on Tuesday, then her schedule for treating patients could be…” In the analysis regarding this question, it states if she operates on Tuesday then she cannot operate on Monday and therefore you can infer she operates on Wednesday and Friday. How is this not a mistaken reversal? The sufficient is operating Monday with necessary not operating Tuesday. The question stem states “If she operates TUESDAY “.
 Adam Tyson
PowerScore Staff
  • PowerScore Staff
  • Posts: 5153
  • Joined: Apr 14, 2011
|
#93747
That's not a Mistaken Reversal, Mswise34, but a valid Contrapositive. If the Necessary Condition (she does not operate on Tuesday) fails to occur (and that double negative of not not operating becomes a positive, so if she DOES operate on Tuesday), then the Sufficient Condition (she operates on Monday) also cannot occur. A Mistaken Reversal in this case would be "she does not operate on Tuesday so she must operate on Monday." That would be a case of the Necessary Condition (not operating Tuesday) occurring and concluding that the Sufficient Condition (operating on Monday) must therefore occur.

An interesting and potentially confusing twist in this game, though, is that it turns out that due to the number of operations in the mornings (she has to do it three times), coupled with the fact that she cannot operate on Saturday, then the Mistaken Reversal described above actually has to be true! If she does not operate on Tuesday, then the only days she can operate are Monday, Wednesday, and Friday. But that's not happening because of the conditional rule. It's a function of the numbers and other restrictions here.

But the issue in Question 11 has nothing to do with a Mistaken Reversal and is not driven entirely by the numbers. It's triggering the contrapositive that proves she cannot operate on Monday. If she did, that would violate the rule!

Get the most out of your LSAT Prep Plus subscription.

Analyze and track your performance with our Testing and Analytics Package.